You are working to earn enough money for a vacation which costs 1200.0 dollars. Your take-home pay is $16.50/hour, but you only work 4-hour shifts each day. How many days will it take for you to earn enough money to pay for your vacation if all the money you earn goes towards your vacation

Answers

Answer 1

Answer:

It will take [tex]18\frac{2}{11} days[/tex] to earn enough money to pay for your vacation.

Step-by-step explanation:

From the question,

The total cost of the vacation is 1200.0 dollars, that is $1200.0

Also, you earn $16.50/hour but you only work for 4 hours each day.

First, we will determine how much you earn in a day.

If you work for 4 hours each day and earn $16.50 / hour; then the amount you earn in a day is

$16.50/hour × 4 hour

= $66.00

Hence, you earn $66.00 dollars each day.

Now, to determine the number of days it will take for you to earn enough money to pay for your vacation, that will be

The cost of the vacation divided by the amount earned each day

Cost of the vacation = $1200.0

Amount earned each day = $ 66.00

Hence, the number of days it will take for you to earn enough money to pay for your vacation is

$1200.0 / $66.00

= [tex]18\frac{2}{11} days[/tex] or 18.18 days

Hence, it will take [tex]18\frac{2}{11} days[/tex] to earn enough money to pay for your vacation.


Related Questions

What is the place value of 3 in 9. 365?​

Answers

Answer:

It is in the tenths place

Step-by-step explanation:

In the number 9.365, 3 is in the tenths place.

place value 3 is Hundred
(9*1000)+ (3*100)+(6*10)+5

i need help please answer quickly this is due tomorrow

Answers

Step-by-step explanation:

elsian 69 pls help me with my latest question i will gove u anything u want pls its due today i am so stuck

Answer:  see below

Step-by-step explanation:

Graph the points and rotate the graph as follows:

90° = 1 turn left180° = 2 turns left (or right)270° = 3 turns left (or 1 right)

or use the following formulas:

90°:  (x, y) → (-y, x)180°: (x, y) → (-x, -y)270°: (x, y) → (y, -x)

1) 180°: (x, y) → (-x, -y)

       D: (1, 4) → (-1, -4)

       E: (7, 6) → (-7, -6)

       F: (8, 3) → (-8, -3)

       G: (2, 1) → (-2, -1)

2) 270°: (x, y) → (y, -x)

       L: (1, 6)  → (6, -1)

       M: (8, 7) → (7, -8)

       N: (5, 2) → (2, -5)

3)  90°: (x, y) → (-y, x)

       Q: (2, 6) → (-6, 2)

       R: (5, 7) → (-7, 5)

       S: (7, 3) → (-3, 7)

       T: (1, 1)  → (-1, 1)

4) 180°: (x, y) → (-x, -y)

       D: (3, 0) → (-3, 0)

       E: (6, 1) → (-6, -1)

       F: (7, -4) → (-7, 4)

       G: (1, -6) → (-1, 6)

     

what is 5/6 divided by 5/9

Answers

Answer:

1 and 1/2

Step-by-step explanation:

Dividing two fractions is the same as multiplying the first fraction by the reciprocal (inverse) of the second fraction.

1.5 as a decimal

1 1/2 as a fraction

Given that every fifteenth person in line will get a coupon for a free box of popcorn at the movies, what is the probability that you don’t get a coupon when you’re in line? Enter a fraction or round your answer to 4 decimal places, if necessary.

Answers

Answer:

We know that every 15th person gets a free box of popcorn.

This depends on order, but nobody knows what is his actual position, so we can assume that:

of 15 people, 1 will get a free box of popcorn.

Then the probability of getting free popcorn is:

1/15

And the negation, (the probability of not getting a free box) is:

1 - 1/15 = 14/15 = 0.93...

The probability of not getting a free box is, [tex]\frac{14}{15}[/tex]

Probability :

Given that, from 15 people, 1 will get a free box of popcorn.

Then the probability of getting free popcorn is:

                                 [tex]P(E)=\frac{1}{15}[/tex]

the probability of not getting a free box is,

           [tex]P=1-P(E)\\\\P=1-\frac{1}{15}=\frac{14}{15}[/tex]

Learn more about the probability here:

https://brainly.com/question/25870256

Josh and Karen live 20 miles apart and at exactly 2 pm, both decide to ride their bikes towards each other's houses so that they can meet on the path somewhere in between. Josh rides at an average speed of 15 miles per hour and Karen rides at an average speed of 10 miles per hour. If Josh takes a 10 minute break after he starts to look at some birds while Karen rides the whole time, how many miles from Karen's house will they meet?

Answers

Answer:

They will meet at a distance 9 miles from Karen's house.

Step-by-step explanation:

Let the distance covered by Josh be [tex]S_{1}[/tex] and the distance covered by Karen be [tex]S_{2}[/tex]

The distance between Josh and Karen's houses is 20 miles. That is, the total distance both of them will cover is 20 miles.

Hence, [tex]S_{1} + S_{2} = 20 miles[/tex]

From the question, if Josh takes a 10 minute break after he starts while Karen rides the whole time, this means

If Karen rides for a total of [tex]t[/tex] mins, then Josh would spend [tex]t-10[/tex] mins for the journey

Also, Josh rides at an average speed of 15 miles per hour

and Karen rides at an average speed of 10 miles per hour

Now, for Josh

Distance = [tex]S_{1}[/tex]

Speed = 15 mile/hour (Covert to mile/minute); (NOTE: 1 hour = 60 mins)

Speed= 0.25 mile/min

Time spent = [tex]t - 10[/tex] mins

From

[tex]Speed =\frac{Distance}{Time}[/tex]

Distance = Speed × Time

Distance = 0.25 mile/min × (t-10) min

[tex]S_{1}[/tex] = 0.25(t-10) mile

[tex]S_{1}[/tex] = 0.25t - 2.5

t = ([tex]S_{1}[/tex] + 2.5) / 0.25 ....... (1)

For Karen,

Distance = [tex]S_{2}[/tex]

Speed = 10 mile/hour (Covert to mile/minute);

Speed= [tex]\frac{1}{6}[/tex] mile/min

Time spent = t

[tex]Speed =\frac{Distance}{Time}[/tex]

Distance = Speed × Time

Then, [tex]S_{2}[/tex] = [tex]\frac{1}{6}[/tex] mile/min × t min

[tex]S_{2}[/tex] = [tex]\frac{1}{6}t[/tex] mile

t = 6[tex]S_{2}[/tex] ......... (2)

Equating equations (1) and (2), we get

([tex]S_{1}[/tex] + 2.5) / 0.25 = 6[tex]S_{2}[/tex]

Then,

([tex]S_{1}[/tex] + 2.5) = 1.5[tex]S_{2}[/tex]

Recall that

[tex]S_{1} + S_{2} = 20 miles[/tex]

Then, [tex]S_{1} = 20 - S_{2}[/tex]

∴[tex]20 - S_{2}[/tex] + 2.5 = 1.5[tex]S_{2}[/tex]

2.5[tex]S_{2}[/tex] = 22.5

[tex]S_{2}[/tex] = 22.5 / 2.5

[tex]S_{2}[/tex] = 9 miles

This means Karen covers a total distance of 9 miles

Hence, they will meet at a distance 9 miles from Karen's house.

HELP I'm stupid and idk what the answer is!

Answers

Answer:

72

Step-by-step explanation:

Answer:

72 m

Step-by-step explanation:

14x4=56

4x4=16

56+16=72

in a survey of 23 college students it was found that 17 were taking an english class 10 were taking a math class and 8 were taking both english and math. how many students were talking a math only class?
a. 6
b. 14
c. 2
d. 4

Answers

Answer:

2

Step-by-step explanation:

out of 10 that were taking math 8 of them were also taking english. 10-8=2

Which equation show the relationship in the table?

Answers

Answer:

hours = 5 x dollars

Step-by-step explanation:

Because, for every hour you earn 5 dollars.

Hours =5 help this hope

-2(m + 7) = 3(m + 2)

Answers

Answer:

m= -4

Step-by-step explanation:

-2m-14=3m+6

-14=5m+6

-20=5m

m= -4

please help, I've tried learning this multiple times but just can't get it down

Answers

1) n times n times n times n times n times n

2) n^6

Brainliest?

The second angle in a triangle is 3° less than twice the first angle. The third angle measure is 8° more than twice the first angle. Find each angle

Answers

Answer: The first angle is 35°, the second angle is 67° and the third angle is 78°.

Step-by-step explanation:

Wee will represent the first angle by f,the second by s, and the third by t.

If gives us the information that the second angle is 3 less than twice the first one, that can be represent by the equation  s= 2f-3  .

The third angle is 8 more than twice the first so it can also be represent by the equation t= 2f + 8 .

Now we know the interior of angles of a triangle adds up to 180 degrees. so see the measures equal 180 to solve for f.

f+ (2f+8) + (2f -3) = 180  Combine like terms on the left side

5f + 5 = 180  

     -5      -5

5f= 175  

f = 35  

So the first angle is 35 degrees.

Second angle.

s=2(35) - 3  

s= 70 -3

s = 67  

The second angle is 67.

Third angle.

t = 2(35) + 8

t= 70 + 8

t= 78

The third angle is 78 degrees.

Now add the angles up to see if they equal 180 degrees.

35 + 67 + 78 = 180  

   180 = 180

POSSIBLE POINTS
Sarah is making hairbows for 6 friends. She needs 1 yards of ribbon for each hairbow. How many total yards of ribbion will she use?

Answers

Answer:

6 yards

Step-by-step explanation:

she need 1 yard of ribbon for each hairbow and 6 friends to make them for

Find the base for the following items.
4 is 1% of what number?

Answers

Answer:

400

Step-by-step explanation:

What is -2/3 plus 1/5?

Answers

Answer:

-7/15

Step-by-step explanation:

Find GCF of both denominators

GCF: 15

Multiply 5 to -2 in -2/3 and 3 to 1 in 1/5

This gives you -10/15 + 3/15

Solve

-7/15

I need the answer to this with the steps, that’d help me out so much thank you❤️.

Answers

Answer: The dimensions of the frame:.

W= 2√10 or about 6.3245 inches

L= 4√10 or about 12.6491 inches

Step-by-step explanation:

This would be so much easier if the area of the box was 72. Then the dimensions of the frame would be 6 × 12. So just a little bit larger and more complicated.

Here the equation is w(2w)=80

2w^2 = 80 (2w is easier to work with than L/2).

w^2=40 (^2 means squared)

√w^2 = √40 square root of both sides: Factor 40. Take out√4

W = √4*10

W= 2√10

L= 2w = 4√10

6.3245 × 12.6491 = 80

The robber stole only 25 pounds
Of quarters out of the coin machine. quarters are weighed in ounces. If there are 16 ounces in 1 pound how many ounces of quarters were stolen?

Answers

The answer is 400 oz

In the figure below, what is the value of xº?
75°
79°
O A. 180°
B. 26°
O C. 64°
O D. 101°

Answers

Answer:

x=26°

B. 26°

Step-by-step explanation:

Triangle Sum theorem

75 + 79 + x =180

154 + x =180

x= 26

Find the volume of oxygen in a room with the dimensions of 144.78 cm x 198.12 cm x 144.78

Answers

Answer:

  4,152,842.53301 cm³ ≈ 4.1528 m³

Step-by-step explanation:

The volume of a rectilinear room of those dimensions will be the product of those dimensions:

  (144.78 cm)(198.12 cm)(144.78 cm) = 4,152,842.53301 cm³

A more appropriate unit for such a volume is cubic meters. There are a million cubic centimeters in a cubic meter, so the volume (to 5 significant digits) is ...

  4.1528 m³

_____

We assume that any oxygen is uniformly distributed in the space, so the volume of oxygen will be the same as the volume of the room.

_____

Additional comment

These dimensions correspond to those of a room that is 57 inches square and 78 inches high.

A unit rate is a rate in which one of the quantities is 1. The unit rate is the same as the constant of proportionality. In our ice cream example, 1 scoop of ice cream costs $2. The unit rate, or unit price, of dollars per scoop is 2. Terrell sells clay pots. He sells 2 clay pots for $6, 3 clay pots for $9, and 4 clay pots for $12. What is the unit rate of dollars per clay pot? A 1 B 2 C 3 D 4

Answers

Answer:

The annswer is 3 for the clay pots

Step-by-step explanation:

Which graph shows the solution to the inequality -3x-7 <20?
+
-10
-5
0
ch
5
10
+
-10
-5
0
5
10
A
0
-5
5
5
10
-10
+
5
10
0
-5
-10

Answers

Answer:

20

Step-by-step explanation:

Answer: 20

Step-by-step explanation:

Need help with these

Answers

Answer:

a. 0.07

b. 0.17

c. 0.333

d. 0.417

A car Is traveling at a speed of 90 kilometers per hour. What is the car's speed in kilometers per minute? How many kilometers will
the car travel in 2 minutes? Do not round your answers.

Answers

[tex]\huge\boxed{\boxed{\bold{1.5\ \bftext{km/min}}}\ \boxed{\bold{3\ \bftext{km}}}}[/tex]

[tex]\hrulefill[/tex]

To convert from kilometers per hour to kilometers per minute, we simply divide by [tex]60[/tex] as there are [tex]60[/tex] minutes in an hour.

[tex]\frac{90}{60}=\frac{90\div30}{60\div30}=\frac{3}{2}=\large\boxed{1.5}[/tex]

To find how far the car will travel in a certain time, just multiply the speed by the time.

[tex]1.5\ \text{km/min}*2\ \text{min}[/tex]

[tex]\large\boxed{3\ \text{km}}[/tex]

A steel woodworking blade is created from a rectangular mold with vertices at (0, 0), (0, 1), (4, 0), and (4, 1). (All coordinates are in centimeters). At each point of the blade, the thickness, in milimeters, is equal to twice the y-coordinate. Find the height of the balance point of the blade.

Answers

Answer:

5.6 cm

Step-by-step explanation:

Given that a steel woodworking blade is created from a rectangular mold with vertices at (0, 0), (0, 1), (4, 0), and (4, 1). (All coordinates are in centimeters). At each point of the blade, the thickness, in milimeters, is equal to twice the y-coordinate. 

The x axis or the horizontal length will be L = (1 - 0)^2 + ( 0 - 0 )^2

L = 1 cm

The y axis or the vertical length will be

L = sqrt ( [ 4-0]^2 + [0-0]^2)

L = Sqrt(16)

L = 4 cm

Given that at each point of the blade, the thickness, in milimeters, is equal to twice the y-coordinate.

That is,

Thickness = 2 × 4 = 8 mm

The height of the balance point of the blade will be;

Height = 8 mm + 4 cm + 8 mm

Height = 0.8 + 4 + 0.8

Height = 5.6 cm

Note that the thickness will be added to both the top and the bottom of the blade.

Identify a reasonable approximation for the solution of this system

Answers

Answer:

Add a picture of the system

Step-by-step explanation:

what is the value of x?
30
45
55
60​

Answers

Answer:

[tex]\boxed {\tt B. \ x=45}[/tex]

Step-by-step explanation:

x and 3x are on a straight line together. Therefore, they are supplementary and must add to 180 degrees.

Add x and 3x, and set that equal to 180.

[tex]x+3x=180[/tex]

Combine like terms on the right side of the equation. Both x and 3x have a variable, so they can be added together.

[tex](x+3x)=180[/tex]

[tex]4x=180[/tex]

Since we want to find x, we have to isolate x. x is being multiplied by 4. The inverse of multiplication is division. Divide both sides of the equation by 4.

[tex]\frac{4x}{4} =\frac{180}{4}[/tex]

[tex]x=\frac{180}{4}[/tex]

[tex]x=45[/tex]

x is equal to 45, so the correct answer is B. 45

In the diagram, we can see that two lines are intersecting and a two pairs of vertically opposite angles are forming. The angle on a line are supplementary to each other (linear pair).

Supplementary angles add upto 180°.

Here, angle TRS and angle TRV are linear pair, so they will have a sum of 180°.

Then,

[tex] \angle TRS + \angle TRV = 180 \degree[/tex]

In the question, given angle TRS = x and angle TRV = 3x

[tex]x + 3x = 180 \degree[/tex]

[tex]4x = 180 \degree[/tex]

Now, divide 4 from both sides because we need to find x.

[tex]x = \frac{180 \degree}{4} [/tex]

[tex]x = 45 \degree[/tex]

It was given that Angle TRS is x

So, Angle TRS = 45°

Option B

#CarryOnLearning

━━━━━━━━━━━━━━━━━━━━

OPEN ENDED QUESTION
Rewrite the following problem using
commutative property of addition: -3 + 7

Answers

Answer:

4

Step-by-step explanation:

7+-3=4

Answer:

4

Step-by-step explanation:

Step 1:

7 + ( -3)

Step 2:

7 - 3

Answer:

4

Hope This Helps :)

You are part of a construction company that is supposed to build houses. An architect has left plans to build houses on each island. Bridges connect the islands. Island A and Island B have a total of 25 houses. Island A and Island C have a total of 10 houses. Island B and Island C have a total of 25 houses. A total of 30 houses is to be built on the three islands. Determine how many houses are to be built on each island.

Answers

Answer:

Island A=5

Island B=20

Island C=5

Step-by-step explanation:

Island AB=25 house

Island AC=10 house

Island BC=25 house

Total =30 house

30-25 to get the house on island C which is 5

30-10 to get the house on island B which is 20

30 -25 to get the house on island A which is 5


5 n/2= (a + b), for a

Answers

Answer:

5n/2 - b =a

Step-by-step explanation:

5n/2 = (a + b)

subtract b from both sides

5n/2 - b = a

Place the numbers in order from least to greatest.
0.24,4 x 10^-2,0.042, 2 x 10^-4,0.004

Answers

Answer:

2x10^-4,0.004,4x10^-2,,0.042,0.24

Step-by-step explanation:

i did the paper

x +2y = 5
5x -y = 3
I need help with my homework please anyone

Answers

Answer:

Step-by-step explanation:

Other Questions
What is the slope of the line in the graph? if it is 9:37 what time would it be a hour and a half later 4. Which expression represents a number greater than 8,000 but smaller than 80,000 An amusement park charges $12 for admission and $1.50 per ride r. Jeremy goes to the park with $42,The equation that represents this situation is 12 + 1,50r = 42. Determine how many rides r Jeremy cango on. Some fossils provide evidence to suggest that Earth's climate has changed over time. What information can scientists obtain from these fossils? temperatures number of species that existed movement of continents cause of mass extinctions why is makyra a loser?? Read the sentence.The tracking party followed their prey to the river, but there they lost the trail, even though they had several bloodhounds with them.What type of sentence is this? the sum of max's age (x) and his sister's age (y) is 8 PLEASE HELP ME OUT!!!What do you think was the most important factor behind the closing of the American fronter the discovery of mineral resources, government and grants and promotions or the completion of the railways ? Use evidence to support your answer . Please Help!! 10 Points!! brainliest answer The price of a computer fell 20% this year. If the computer now costs $750, how much did it cost last year?(step-by-step if possible) Given M is the midpoint of PQ with PM = 1/2x + 5 and PQ = 26, find x.Yeah Katie and her family are moving to a different community. What is the best way to learn about the various physical activities available where she is moving? Can someone help with this Select the correct answer.Read the excerpt from the passage.In the first frantic greetings lavished on himself as a noted sufferer under the overthrown system, it had been accorded to him to have Charles Darnay brought before the lawless Court and examined.Which type of phrase is the underlined portion of the sentence?A. adverbial phraseB. verb phraseC. adjectival phraseD. absolute phrase why were southerners in favor of texas annexation and northeners opposed to it?Explaination:helpppppp I need asap! Find the domain and range of f(x)=|x|+ cos xI need help ASAP URGENT!!! James has $40 to spend. He wants to buy two tickets to a show. He has to spend $7 to get to where tickets are sold. The following inequality can be used to find out the highest price James can afford to pay for each ticket. 2r +7 23.5 Or> 16.5 2gallon container of window cleaner costs $15.04. What is the price per cup? steve cycled 35 miles